0
$\begingroup$

Here is my setup. I have two sets of DNA sequences, one bound by a protein (transcription factor), another not bound by the protein. Let's say I have two candidate motifs, or DNA patterns the protein might tend to bind to. The prevalence ratio of motif 1 is the proportion of bound sequences containing motif 1 divided by the proportion of unbound sequences containing motif 1. Similarly for motif 2. How do I test whether the prevalence ratio of motif 1 is greater than the prevalence ratio of motif 2?

More mathematically, I want to test whether $$\frac{P(M_1|B)}{P(M_1|\bar B)} > \frac{P(M_2|B)}{P(M_2|\bar B)}$$ because by Bayes' rule, $\frac{P(B|M)}{P(\bar B|M)} = \frac{P(M|B)P(B)}{P(M|\bar B)P(\bar B)}$, i.e. the odds of being bound goes up by a factor of the prevalence ratio upon observing the motif. So this is asking whether motif 1 increases the odds of being bound more than motif 2.

More generally we observe:

  • $n$ points with the condition (set 1)
  • $m$ points without the condition (set 2)
  • $k_1$ points in set 1 with marker 1
  • $k_2$ points in set 1 with marker 2
  • $\ell_1$ points in set 2 with marker 1
  • $\ell_2$ points in set 2 with marker 2

Using this we want to test whether $$ \frac{p_1}{q_1} > \frac{p_2}{q_2}$$ where $p_1$ is the probability a datapoint with the condition has marker 1, $q_1$ is the probability a datapoint without the condition has marker 1, $p_2$ is the probability a datapoint with the condition has marker 2, and $q_2$ is the probability a datapoint without the condition has marker 2.

Another example could be, the condition is lung cancer, and marker 1 is smoking, and marker 2 is some genetic condition. So we want to say whether smoking increases the odds of lung cancer more than the genetic condition does, by finding a random set of individuals with lung cancer and a random set of individuals without lung cancer, and comparing the prevalence ratios of smoking / the genetic condition among these sets.

$\endgroup$
2
  • $\begingroup$ Does this thread stats.stackexchange.com/questions/462951/… answer your question? $\endgroup$ Commented Mar 24, 2024 at 9:33
  • $\begingroup$ That thread is asking for whether the proportion of a trait in one population is higher than the proportion in another population, which is handled by Fisher's exact test. I'm asking for whether the ratio of the proportion of a trait in one population to the proportion in another population is greater than the ratio for another trait. $\endgroup$ Commented Mar 24, 2024 at 17:09

0

Your Answer

By clicking “Post Your Answer”, you agree to our terms of service and acknowledge you have read our privacy policy.

Start asking to get answers

Find the answer to your question by asking.

Ask question

Explore related questions

See similar questions with these tags.